Difference between revisions of "2006 AMC 12A Problems/Problem 12"

m (Solution: typos)
m (Solution: should use show preview button more often... anyway, more typos)
Line 7: Line 7:
  
 
== Solution ==
 
== Solution ==
The sum of the [[consecutive]]ly increasing [[integers]]s from 3 to 20 is <math> \frac{1}{2}(18)(3+20) = 207 </math>. However, the 17 [[intersection]]s between the rings must also be subtracted, so we get <math> 207 - 2(17) = 173 \Rightarrow B </math>.
+
The sum of the consecutively increasing [[integer]]s from 3 to 20 is <math> \frac{1}{2}(18)(3+20) = 207 </math>. However, the 17 [[intersection]]s between the rings must also be subtracted, so we get <math> 207 - 2(17) = 173 \Rightarrow B </math>.
  
 
== See also ==
 
== See also ==

Revision as of 19:41, 31 January 2007

Problem


An image is supposed to go here. You can help us out by creating one and editing it in. Thanks.


A number of linked rings, each 1 cm thick, are hanging on a peg. The top ring has an outside diameter of 20 cm. The outside diameter of each of the outer rings is 1 cm less than that of the ring above it. The bottom ring has an outside diameter of 3 cm. What is the distance, in cm, from the top of the top ring to the bottom of the bottom ring?

$\mathrm{(A) \ } 171\qquad \mathrm{(B) \ } 173\qquad \mathrm{(C) \ } 182\qquad \mathrm{(D) \ } 188$$\mathrm{(E) \ }  210$

Solution

The sum of the consecutively increasing integers from 3 to 20 is $\frac{1}{2}(18)(3+20) = 207$. However, the 17 intersections between the rings must also be subtracted, so we get $207 - 2(17) = 173 \Rightarrow B$.

See also


{{{header}}}
Preceded by
Problem 11
AMC 12A
2006
Followed by
Problem 13